PDE with variable boundary condition

Click For Summary
The discussion revolves around solving a partial differential equation (PDE) with variable boundary conditions using the combination method. The user encounters difficulties with the second boundary condition, which does not transform properly to the new variable. Feedback indicates that the combination method is unsuitable for this type of boundary condition, as it cannot express the required derivatives as functions of the new variable. The suggested alternative is to use the Laplace transform method to address the problem effectively. The conversation highlights the challenges of variable boundary conditions in PDEs and the importance of selecting appropriate solution methods.
jafanasim
Messages
2
Reaction score
0

Homework Statement



I am trying to solve this PDE with variable boundary condition, and I want to use combination method. But I have problem with the second boundary condition, which is not transformed to the new variable. Can you please give me some advise?

Homework Equations



(∂^2 T)/(∂x^2 )=1/∝ ∂T/∂t
IC: T(x,0) = Ti
BC1: T(x→∞, t) = Ti
BC2: -k ∂T/∂x|x=0 = h[T∞ - T(0,t)]

The Attempt at a Solution



The combination variable I chose is η= x/((4∝t)^(1⁄2))

My work is attached in a PDF file, please take a look at it. The highlighted boundary condition is the problem.

Thank you
 

Attachments

Physics news on Phys.org
That method does not work with that sort of boundary condition: you can't express \partial \eta/\partial x as a function of \eta.
 
pasmith said:
That method does not work with that sort of boundary condition: you can't express \partial \eta/\partial x as a function of \eta.

Thank you. What method should I choose? Laplace?
 
jafanasim said:
Thank you. What method should I choose? Laplace?

That would be the obvious choice.
 
Question: A clock's minute hand has length 4 and its hour hand has length 3. What is the distance between the tips at the moment when it is increasing most rapidly?(Putnam Exam Question) Answer: Making assumption that both the hands moves at constant angular velocities, the answer is ## \sqrt{7} .## But don't you think this assumption is somewhat doubtful and wrong?

Similar threads

  • · Replies 5 ·
Replies
5
Views
1K
Replies
1
Views
2K
Replies
6
Views
2K
  • · Replies 1 ·
Replies
1
Views
2K
Replies
1
Views
2K
  • · Replies 4 ·
Replies
4
Views
4K
  • · Replies 4 ·
Replies
4
Views
2K
  • · Replies 1 ·
Replies
1
Views
1K
  • · Replies 1 ·
Replies
1
Views
1K
  • · Replies 3 ·
Replies
3
Views
3K